Difference between revisions of "2022 AMC 8 Problems/Problem 8"

(Created page with "==Problem== COPY AND PASTE THE PROBLEM HERE. ==Solution== ==See Also== {{AMC8 box|year=2022|num-b=7|num-a=9}} {{MAA Notice}}")
 
(Problem: added problem)
Line 1: Line 1:
 
==Problem==
 
==Problem==
COPY AND PASTE THE PROBLEM HERE.
+
What is the value of <cmath>\frac{1}{3}\cdot\frac{2}{4}\cdot\frac{3}{5}\cdots\frac{18}{20}\cdot\frac{19}{21}\cdot\frac{20}{22}?</cmath>
 +
 
 +
<math>\textbf{(A)} ~\frac{1}{462}\qquad\textbf{(B)} ~\frac{1}{231}\qquad\textbf{(C)} ~\frac{1}{132}\qquad\textbf{(D)} ~\frac{2}{213}\qquad\textbf{(E)} ~\frac{1}{22}\qquad</math>
  
 
==Solution==
 
==Solution==

Revision as of 11:50, 28 January 2022

Problem

What is the value of \[\frac{1}{3}\cdot\frac{2}{4}\cdot\frac{3}{5}\cdots\frac{18}{20}\cdot\frac{19}{21}\cdot\frac{20}{22}?\]

$\textbf{(A)} ~\frac{1}{462}\qquad\textbf{(B)} ~\frac{1}{231}\qquad\textbf{(C)} ~\frac{1}{132}\qquad\textbf{(D)} ~\frac{2}{213}\qquad\textbf{(E)} ~\frac{1}{22}\qquad$

Solution

See Also

2022 AMC 8 (ProblemsAnswer KeyResources)
Preceded by
Problem 7
Followed by
Problem 9
1 2 3 4 5 6 7 8 9 10 11 12 13 14 15 16 17 18 19 20 21 22 23 24 25
All AJHSME/AMC 8 Problems and Solutions

The problems on this page are copyrighted by the Mathematical Association of America's American Mathematics Competitions. AMC logo.png